Calculus: Limits - Solving for Lim[f(x) + 2g(x)]

  • Thread starter Thread starter Ris Valdez
  • Start date Start date
  • Tags Tags
    calculus limits
Click For Summary
The limit of the expression lim[f(x) + 2g(x)] is calculated using the known limits of f(x) and g(x), where lim f(x) = -4 and lim g(x) = 6 as x approaches infinity. By substituting these values, the limit simplifies to -4 + 2(6), resulting in a final limit of 8. The discussion confirms that the calculations are correct and emphasizes the properties of limits that support this conclusion. Participants express reassurance and clarify the reasoning behind the limit calculation. The final answer is confirmed as correct, reinforcing the understanding of limit properties.
Ris Valdez
Messages
9
Reaction score
0

Homework Statement


Given that
lim f(x) = -4 and lim g(x) = 6
(All limits x --> +infinity)

Find the limit
lim [f(x) + 2g(x)]

Homework Equations



The Attempt at a Solution


So I substituted the values of f(x) and g(x) in the equation

=[(-4) + 2(6)
the limit is = 8

Did I do it right?
 
Physics news on Phys.org
Ris Valdez said:
Did I do it right?
Yep
 
Nathanael said:
Yep
Thanks! It was a wiley assignment and I thought I did it wrong xD
 
Ris Valdez said:
Thanks! It was a wiley assignment and I thought I did it wrong xD

What else could the limit possibly be?
 
PeroK said:
What else could the limit possibly be?
Sorry! I was just making sure.
 
You should know from basic properties of limits that
1) For any constant A, as long as lim f(x) exists, then so does lim Af(x) and the limit is A(lim f(x)).
2) As long as lim f(x) and lim g(x) exist, then so does lim f(x)+ g(x) and the limit is lim f(x)+ lim g(x).

Those two together give the result you want.
 
Question: A clock's minute hand has length 4 and its hour hand has length 3. What is the distance between the tips at the moment when it is increasing most rapidly?(Putnam Exam Question) Answer: Making assumption that both the hands moves at constant angular velocities, the answer is ## \sqrt{7} .## But don't you think this assumption is somewhat doubtful and wrong?

Similar threads

  • · Replies 6 ·
Replies
6
Views
2K
  • · Replies 6 ·
Replies
6
Views
1K
  • · Replies 2 ·
Replies
2
Views
1K
  • · Replies 2 ·
Replies
2
Views
1K
  • · Replies 5 ·
Replies
5
Views
2K
  • · Replies 7 ·
Replies
7
Views
1K
Replies
2
Views
2K
  • · Replies 19 ·
Replies
19
Views
2K
  • · Replies 2 ·
Replies
2
Views
1K
  • · Replies 1 ·
Replies
1
Views
2K